LSAT and Law School Admissions Forum

Get expert LSAT preparation and law school admissions advice from PowerScore Test Preparation.

 Administrator
PowerScore Staff
  • PowerScore Staff
  • Posts: 8917
  • Joined: Feb 02, 2011
|
#23107
Complete Question Explanation

Method of Reasoning. The correct answer choice is (C)

The politician idiotically argues that since a 100% tax rate would stymie all business and therefore all tax revenue, then the lower the tax rate, the higher the tax revenue.

The economist points out the obvious flaw in the politician's reasoning, which is that according to the politician's principle a tax rate of 0% ought to produce the most tax revenue.

You are asked to identify the economist's strategy, so you should focus on the fact that the economist demonstrates the absurdity of the politician's principle.

Answer choice (A) The economist demonstrates the inanity of the politician's principle, rather than offering any alternative.

Answer choice (B) The economist offers no such evidence.

Answer choice (C): This is the correct answer choice. The limiting case is the most extreme case, which in this case is that of 0%. The economist refers to the 0% case to demonstrate the absurdity of the politician's principle.

Answer choice (D) The economist's argument does have the effect of undermining our belief in the politician's ability to engage in rational thought; however, that is not how the economist makes his point. The difference between the economist's argument and a source attack is that in a source attack the conclusion is based on rejecting the source (Source bad → argument bad), but in this case we tend to reject the source because the argument was so bad (Argument bad → source bad). Since those concepts are related through a Mistaken Reversal, they are not equivalent to each other.

Answer choice (E) The economist attacks the politician's principle, which is the politician's conclusion. The politician's premise was that a 100% tax rate would result in zero tax revenue, and the economist leaves that somewhat sound commonsense premise alone.
 egarcia193
  • Posts: 41
  • Joined: Jun 25, 2017
|
#37805
I chose A and I am confused about how C is correct, it talks about a limited case and from that I understand it to be saying that it is an example which would not be true because he gives no specific example in his argument, and furthermore the general principle A is talking about is the principle the politician uses in an attempt to prove his point which is then used against him to prove he is false So I don't really understand why C is correct ?
 Francis O'Rourke
PowerScore Staff
  • PowerScore Staff
  • Posts: 471
  • Joined: Mar 10, 2017
|
#37922
There is an important distinction between a limiting and a limited case. The former refers to an extreme or radical case. The latter refers to a specific case.

Although you can describe the politician's argument as using a general principle, the economist does not state an incompatible general principle. We are asked to describe a strategy used by the economist, so for answer choice (A) to be correct, we would need to find a new principle introduced in this speaker's reply.
 Naj
  • Posts: 8
  • Joined: Dec 16, 2019
|
#73413
I am having a really hard time understanding this questions. I was stuck between D and E. The concept of "attack" seemed like a strategy here . I am also confused with the word choice "principle" is it a method of approach? or some sort of requirement?

Here is why i was confused with D and E:
D- Undermining the credibility of Politician by making the extreme argument that "tax of 0 percent would generate the maximum revenue" to present doubt and undermine the fact that Politician's premise is merely an exaggeration.

E - while the Economist states clearly that the conclusion is not correct but it takes the premise of the Politician and uses it as an extreme example to doubt the reasoning of the Politician.

What am I doing wrong here? is it my understanding of the argument or my understanding of the answer choices.

Thank you
 Adam Tyson
PowerScore Staff
  • PowerScore Staff
  • Posts: 5153
  • Joined: Apr 14, 2011
|
#73420
You're right that the Economist attacks the argument made by the Politician, Naj, but just saying the strategy is an attack isn't specific enough. I think the issue you are having is mainly with the answer choices, so let's take a look at what your two contenders mean.

Answer D describes what we call a Source Argument, which you may also know as an ad hominem attack. That's where you attack the person who made the argument, rather than attacking the argument itself. In its crudest form it's basically "the politician is wrong because he's stupid." A source argument ignores the evidence and focuses instead on the characteristics of the opposing party - their credentials, their intelligence, their motives, etc. That didn't happen in this case, because the Economist responded only to the claims made in the argument and said nothing at all about the Politician on any personal level. That makes D a wrong answer - it describes something that did not happen in this case. The Economist never questioned the Politician's understanding of economics, but only pointed out an absurd consequence of the Politician's conclusion.

Answer E is about attacking a premise, which is at least about addressing the argument made by the Politician. The problem with it is that the Economist never attacked the Politician's only premise, which was that a tax rate of 100% would generate no revenue. Instead, the Economist undermined the Politician's conclusion, by showing that it had an absurd consequence when taken to the extreme. The Economist might completely agree with the Politician's premise!

As to the word "principle" on the LSAT, think of it as being synonymous with "rule" or "guideline." A general principle is a broad rule, like "a reduction in supply leads to an increase in demand" or "one should brush their teeth after every meal." You'll see that term pop up in a lot of different question types and answer choices. When you do, ask yourself what the rule is, if there even is one. In this case, the Politician's rule is his conclusion - the lower the tax rate, the more revenue it will generate. The Economist applied that rule to the lowest possible tax rate of 0% and showed that the conclusion made no sense in that context.

Discussions of Principles can be found in our course materials (Lesson 8 in our full length course) and in the LR Bible. Methods of Reasoning (which is this question type) are covered in the Bible and in Lesson 7 in our full length course. You can also find some helpful info about Flaws in the Reasoning, such as Source Arguments, in our blog and our podcast, both of which did multi-part series on common flaws.
 Naj
  • Posts: 8
  • Joined: Dec 16, 2019
|
#73444
Thank you so much Adam, I now have a better idea.
 Nicholas Noyes
  • Posts: 38
  • Joined: Feb 07, 2020
|
#74252
Correct me if I am wrong, so if answer E attacked the premise and mentioned "economic activity" then that answer could have been correct? I still am having a difficult time understanding why C is correct, could you elaborate a little further please? What does question C mean "in the limiting case, leads to an absurdly false conclusion"...the language is confusing me.
 Jeremy Press
PowerScore Staff
  • PowerScore Staff
  • Posts: 1000
  • Joined: Jun 12, 2017
|
#74277
Hi Nicholas,

Thanks for your questions! In order for answer choice E to be correct, the economist would've had to say something that would give a "reason to doubt the truth of a premise." The only premise in the politician's argument is that "[a] government that taxes incomes at a rate of 100 percent will generate no revenue because all economic activity will cease." To give a reason to doubt that premise, the economist would have to say something that would suggest the government could still generate revenue even if it taxed people at a rate of 100 percent. Since the economist doesn't do that (doesn't refer to anything that specifically addresses the 100 percent tax rate), answer choice E must be wrong.

The answer choice C language of "limiting case" means "the case at the most extreme limit of possible values." Taxing at 0% is the lowest possible value for the tax rate. At that rate, the conclusion the politician reaches (that the revenue government generates will be at its highest) is absurdly false, because if the government taxes at 0%, it will generate zero revenue (not maximum revenue).

Does that clear up that answer?

Thanks!

Jeremy

Get the most out of your LSAT Prep Plus subscription.

Analyze and track your performance with our Testing and Analytics Package.